¿Cómo obtener la forma explícita de la función de Green de la ecuación de Klein-Gordon?

La definición de la función de Green para la ecuación de Klein-Gordon dice:

( t 2 2 + metro 2 ) GRAMO ( X , t ) = d ( t ) d ( X )
Según estos recursos:

  1. La función de Green para la ecuación no homogénea de Klein-Gordon , la función de Green se ve así:

GRAMO ( X , t ) = θ ( t ) 2 π d ( t 2 | X | 2 ) metro 2 π θ ( t | X | ) j 1 ( metro t 2 | X | 2 ) metro t 2 | X | 2

  1. De la wikipedia, hay varios tipos de propagadores , que dependen de la elección del contorno. Se parece mucho al de arriba, pero aún no es idéntico a mí. Por ejemplo, la función de green avanzado y retardado se parece mucho a la expresión anterior.

Quiero derivar la forma explícita de la función verde mencionada anteriormente , esto es lo que probé:

Asumir:

GRAMO ( X , t ) =   d 3 pag   d ω mi i ( pag X ω t ) GRAMO ( pag , ω )

Sustituyendo a la ecuación de Klein-Gordon se obtiene:

GRAMO ( pag , ω ) = 1 ( 2 π ) 4 1 ω 2 pag 2 metro 2

Entonces:

GRAMO ( X , t ) = 1 ( 2 π ) 4   d 3 pag   d ω mi i ( pag X ω t ) 1 ω 2 pag 2 metro 2

En la fórmula anterior, los polos están en el eje real, para obtener una respuesta finita, es necesario manipular el polo ligeramente alejándolo del eje real. En la elección de Feynman, uno tiene el poste izquierdo ligeramente arriba y el poste derecho ligeramente hacia abajo, así:ingrese la descripción de la imagen aquí

Entonces nosotros tenemos:

GRAMO ( X , t ) = 1 ( 2 π ) 4   d 3 pag   d ω mi i ( pag X ω t ) 1 ω 2 pag 2 metro 2 + i ϵ integrar sobre  ω = θ ( t ) i ( 2 π ) 3   d 3 pag mi i pag X mi i pag 2 + metro 2 t 2 pag 2 + metro 2 + θ ( t ) i ( 2 π ) 3   d 3 pag mi i pag X mi i pag 2 + metro 2 t 2 pag 2 + metro 2 integrar sobre  ϕ  y  θ = i ( 2 π ) 2 θ ( t ) | X | 0 pag pecado ( pag | X | ) mi i pag 2 + metro 2 t pag 2 + metro 2 + i ( 2 π ) 2 θ ( t ) | X | 0 pag pecado ( pag | X | ) mi i pag 2 + metro 2 t pag 2 + metro 2

La derivación anterior parece no tener ningún defecto y no sé cómo proceder. pag integral, y no puedo ver el parecido de la fórmula actual con la fórmula cerrada dada en los recursos 1 y 2.

Integrar pag , encontré que esta integral del libro podría ayudar:

0 mi β γ 2 + X 2 porque b X = β γ γ 2 + β 2 k 1 ( γ β 2 + b 2 ) ,  con:  R mi β > 0 , R mi γ > 0

editar2

---para abordar la respuesta actualizada de @Solenodon Paradoxus

La respuesta sugería rotar el contorno en sentido contrario a las agujas del reloj ( ω = i ω ), por lo tanto:

GRAMO F ( pag , ω ) = 1 ω 2 | pag | 2 metro 2 + i ϵ = 0 d L mi L ( ω 2 + | pag | 2 + metro 2 i ϵ )

Conecta lo anterior GRAMO F ( pag , ω ) en la integral de cuatro impulsos:

GRAMO F ( X , t ) = i ( 2 π ) 4 0   d L mi ( metro 2 i ϵ ) L d ω d 3 pag mi L ω 2 + t ω mi L pag 2 + i X pag = i dieciséis π 2 0   d L mi ( metro 2 i ϵ ) L 1 L 2 mi τ 2 4 L  con  τ 2 = t 2 | X | 2

Desde ahora ϵ no es importante realizar la integral anterior, simplemente omítala.

GRAMO F ( X , t ) = i dieciséis π 2 0 d L mi metro 2 L 1 L 2 mi τ 2 4 L = i metro 2 dieciséis π 2 0 d ξ mi ξ mi metro 2 τ 2 4 ξ 1 ξ 2
Cuando τ 2 > 0 , el integrando diverge en ξ 0 , la integral no se puede realizar.

Cuando τ 2 < 0 , tenemos:

i metro 4 π 2 τ 2 k 1 ( metro τ 2 )
dónde k 1 es la función de Bessel modificada, esta es exactamente la forma del propagador de Feynmann cuando τ 2 < 0 .

Pregunta: Aunque nuestro resultado triunfa por un lado, ¿qué pasa con el otro lado ( τ 2 > 0 )? ¿Cómo podemos obtenerlo del procedimiento anterior? ¿En qué paso excluimos esta posibilidad?

Respuestas (2)

Las funciones de Green no son únicas. Cualquier solución de eso satisface la ecuación homogénea,

( t 2 2 + metro 2 ) F = 0
en la región de interés se puede agregar a la función de Green sin estropear la ecuación no homogénea. La parte homogénea debe, por lo tanto, estar determinada por condiciones de contorno. Puede obtener la función de Bessel como parte de la versión de Feynman de la función de Green resolviendo la función de Green estacionaria con un espacio de 4 dimensiones, es decir:
( 1 r 3 r [ r 3 r ] + metro 2 ) GRAMO ( r ) = i Ω 4 r 3 d ( r )
y continuando analíticamente hasta el tiempo imaginario. Dentro del cono de luz delantero, donde el argumento de k es imaginario, da una función de Hankel del primer tipo:
metro 4 π 2 τ 2 k 1 ( i metro τ 2 ) = metro 8 π τ 2 H 1 ( 2 ) ( metro τ 2 ) = metro 8 π τ 2 [ j 1 ( metro τ 2 ) i Y 1 ( metro τ 2 ) ] ,
con j a y Y a las funciones de Bessel de primera y segunda clase, respectivamente.

Debido a que tanto el operador como la parte no homogénea son reales, la parte imaginaria de la función de Green debe ser una solución a la ecuación homogénea y la parte real debe resolver la no homogénea. Eso es:

( t 2 2 + metro 2 ) Soy { metro 4 π 2 τ 2 k 1 ( i metro τ 2 ) } = 0 ,   a norte d ( t 2 2 + metro 2 ) Re { metro 4 π 2 τ 2 k 1 ( i metro τ 2 ) } = d 4 ( X ) ,
entonces la función de Green es la parte real de esa expresión.

Características que quedan por mostrar en este punto de esta publicación: que la división en propagadores retardados y avanzados es válida (necesaria para preservar la causalidad), y el límite de masa cero del propagador da el límite correcto. La energía inyectada por el impulso no se puede rastrear porque es infinita.

Tal vez la segunda ecuación debería escribirse como:
[ 1 r 3 r r 3 r + metro 2 ] GRAMO ( r ) = d ( r )
?
Buena captura en los paréntesis fuera de lugar. Sin embargo, el lado derecho solo necesitaba una corrección de signo. Es una función delta de 4 dimensiones, por lo que debe tener el ángulo sólido de 4 dimensiones y el factor de r 3 .
Sigo pensando que el lado derecho debería cambiar, la ecuación original es:
( t 2 2 + metro 2 ) GRAMO ( X , t ) = d ( t ) d ( X )
Quieres hacer una sustitución t i w , la ecuación anterior se convierte en:
( 4 2 + metro 2 ) GRAMO ( r ) = d ( i w ) d ( X )
Ahora 4 2 = 1 r 3 r ( r 3 r ) + L 2 r 2 , dónde L 2 es el operador que diferencia la parte angular. si asumimos GRAMO ( r ) = GRAMO ( r ) , L 2 actuar en él solo da cero. Por lo tanto, la ecuación se convierte en:
( 1 r 3 r ( r 3 r ) + metro 2 ) GRAMO ( r ) = d ( i w ) d ( X )
Una pregunta , en esta etapa, creo que estamos considerando w ser una variable real. Sin embargo, cuando se trata de d ( i w ) (¿bien definida?), tenemos la identidad que d ( i w ) = i d ( w ) ? por lo tanto, el lado derecho probablemente debería cambiarse como i d ( r ) ?
De hecho, me perdí el i en el cambio al espacio euclidiano de 4 dimensiones. Y, sí, tenemos d ( i w ) = i d ( w ) .
Que pasa con el primer comentario, que creo 1 r 3 Ω 4 es innecesario
Ah, esa parte es incorrecta. Una función delta tiene que obedecer a la identidad:
d ( X ) d V = 1 ,
no importa la dimensionalidad de X . Eso significa que cuando cambias de variable, por ejemplo a algún sistema de coordenadas esféricas, la función delta debe dividirse por el determinante del jacobiano para cancelar el factor asociado con el elemento de volumen. En este caso, d V = Ω 4 r 3 d r .
He publicado una prueba completa como Preguntas y respuestas: physics.stackexchange.com/questions/280212/… .

Podría intentar usar el método de tiempo adecuado:

1 pag 2 metro 2 + i ε = 0 d L Exp [ L ( pag 2 metro 2 + i ε ) ] .

El truco consiste en hacer primero las integrales de cantidad de movimiento (gaussianas) y luego proceder con la integral sobre L . Esto debería dar la función de Bessel. Déjame saber en los comentarios si tienes más preguntas.

ACTUALIZACIÓN: ¿Cómo manejar la firma de Lorentz en el propagador?

Nos interesa la siguiente integral:

D ( X ) = d 4 pag ( 2 π ) 4 mi i pag m X m pag 2 metro 2 + i ε = d ω 2 π d 3 pag ( 2 π ) 3 mi i ω t mi i pag X ω 2 pag 2 metro 2 + i ε .

Ahora podemos rotar el contorno de integración en el plano complejo de tal manera que ningún polo lo transite. Rotaré el contorno de tu publicación original 90 grados en sentido contrario a las agujas del reloj:

D ( X ) = i d ω 2 π d 3 pag ( 2 π ) 3 mi ω t mi i pag X ω 2 + pag 2 + metro 2 .

También he pasado a la nueva variable de integración: ω = i ω . Ahora el denominador siempre es positivo y puedes usar el método de tiempo adecuado. Todas las integrales serán gaussianas y convergentes.

ACTUALIZACIÓN 2: El i ε nos permite girar el contorno en el sentido contrario a las agujas del reloj, pero prohíbe otras transformaciones (similares), por ejemplo, girarlo en el sentido de las agujas del reloj, como hiciste tú. Esto se debe a que ningún polo debe transitar por el contorno durante su deformación. Por eso, ω = i ω , no i ω . Las transformaciones suaves del contorno no cambian la integral mientras ningún polo la transite.

Entonces, el papel jugado por i ε es determinar cómo se va a transformar la integral en la integral euclidiana. En el caso euclidiano (después de rotar el contorno), puede omitir el i ε .

ACTUALIZACIÓN 3: ya que hemos rotado el contorno, ω se vuelve imaginario y ω = i ω es real (el objetivo de introducir ω′ en realidad). Por eso, ω 2 + pag 2 + metro 2 no puede ser menor que cero!

ACTUALIZACIÓN 4: con respecto a su respuesta final, sospecho que si toma ω = i ω y no i ω (no se le permite pasar por el poste, ¿recuerda?) entonces obtendrá la respuesta correcta. No estoy seguro, por supuesto, pero eso es lo que haría.

Obtienes diferentes funciones de Green. Todo depende de cómo el contorno camina alrededor de los polos, realmente. O sobre dónde enchufar el i ε , si lo desea.

ACTUALIZACIÓN 5: Cómo manejar el τ 2 > 0 ¿caso?

Bueno, esto es lo que se me ocurrió. Podría rotar los contornos de las tres integrales sobre pag en lugar del que se acabó ω . Después de cálculos similares, llegará a la segunda función de Hankel H 1 ( 2 ) . Alternativamente, dado que usamos números complejos en todas partes, podríamos simplemente hacer la continuación analítica del resultado para τ 2 < 0 , lo que nos daría exactamente la misma función de Hankel.

Todavía estoy luchando por encontrar una explicación clara de por qué aparece un símbolo delta adicional (no es como si no pudiera aparecer, ya que solo hemos calculado nuestro propagador para τ 2 > 0 y τ 2 < 0 por ahora).

¿Has asumido que pag 2 metro 2 > 0 ? Si se escribe como el tuyo, al hacer integral sobre cantidad de movimiento, los coeficientes delante de la pag 2 debe ser al menos negativa en la parte real.
desde el ω 2 pag 2 persistir siempre, integrar los cuatro impulsos parece imposible usando la integral gaussiana.
@buzhidao en mi notación, pag 2 = pag 0 2 pag 2 . No, pag 2 metro 2 no siempre es mayor que cero. Puede utilizar la continuación analítica de la integral gausiana.
Vea mi primer comentario, creo que la integral gaussiana puede definirse correctamente solo si la parte real del coeficiente del cuadrado es menor que cero. De lo contrario, simplemente no puedes hacer eso, es divergente.
@buzhidao si considera el caso euclidiano, el propagador (euclidiano) es proporcional a ( pag 2 + metro 2 ) 1 y todas las integrales gaussianas están bien definidas. Luego, puede obtener el propagador lorentziano por la continuación analítica (pasando a los intervalos de tiempo imaginarios) tal como lo explicó en su pregunta. Alternativamente, puede pensar en las integrales gaussianas definidas por continuación analítica incluso cuando el coeficiente delante del término cuadrado es positivo.
@buzhidao He actualizado mi respuesta con detalles sobre cómo manejar correctamente la firma de Lorentz, por favor, eche un vistazo. Como esta es una pregunta de tarea y ejercicios, no voy a presentar el cálculo completo. Tendrá que realizar los detalles del cálculo usted mismo, lo siento. Si tiene más preguntas, por favor hágamelo saber.
Agregué la derivación y enumeré mis preocupaciones, por favor, eche un vistazo.
@buzhidao He actualizado mi respuesta una vez más, por favor, eche un vistazo.
Su explicación es bastante clara, entendí mal la deformación del contorno como una sustitución de variables, por lo tanto, me preocupa el problema de la divergencia sobre el método de tiempo adecuado. Ahora actualicé el cálculo y planteé una nueva inquietud, eche un vistazo.
@buzhidao Tendré que pensar en esto. Estaremos en contacto dentro de 1-2 días.
@buzhidao Mientras pienso en esto, por favor mire mi actualización #5.